Difference between revisions of "2018 AMC 12B Problems/Problem 23"

(Created page with "== Problem == Ajay is stading at point <math>A</math> near Pontianak, Indonesia, <math>0^\circ</math> latitude and <math>110^\circ \text{ E}</math> longitude. Billy is standi...")
 
(Problem)
Line 1: Line 1:
 
== Problem ==
 
== Problem ==
  
Ajay is stading at point <math>A</math> near Pontianak, Indonesia, <math>0^\circ</math> latitude and <math>110^\circ \text{ E}</math> longitude. Billy is standin at point <math>B</math> near Big Baldy Mountain, Idaho, USA, <math>45^\circ \text{ N}</math> latitude and <math>115^\circ \text{ W}</math> longitude. Assume that Earth is a perfect sphere with center <math>C</math>. What is the degree measure of <math>\angle ACB</math>?
+
Ajay is standing at point <math>A</math> near Pontianak, Indonesia, <math>0^\circ</math> latitude and <math>110^\circ \text{ E}</math> longitude. Billy is standin at point <math>B</math> near Big Baldy Mountain, Idaho, USA, <math>45^\circ \text{ N}</math> latitude and <math>115^\circ \text{ W}</math> longitude. Assume that Earth is a perfect sphere with center <math>C</math>. What is the degree measure of <math>\angle ACB</math>?
  
 
<cmath>\textbf{(A) }105 \qquad
 
<cmath>\textbf{(A) }105 \qquad

Revision as of 05:41, 18 February 2018

Problem

Ajay is standing at point $A$ near Pontianak, Indonesia, $0^\circ$ latitude and $110^\circ \text{ E}$ longitude. Billy is standin at point $B$ near Big Baldy Mountain, Idaho, USA, $45^\circ \text{ N}$ latitude and $115^\circ \text{ W}$ longitude. Assume that Earth is a perfect sphere with center $C$. What is the degree measure of $\angle ACB$?

\[\textbf{(A) }105 \qquad \textbf{(B) }112\frac{1}{2} \qquad \textbf{(C) }120 \qquad \textbf{(D) }135 \qquad \textbf{(E) }150 \qquad\]

Solution

Suppose that Earth is a unit sphere with center $(0,0,0).$ We can let \[A=(1,0,0), B=\left(-\frac{1}{2},\frac{1}{2},\frac{\sqrt 2}{2}\right).\]The angle $\theta$ between these two vectors satisfies $\cos\theta=A\cdot B=-\frac{1}{2},$ yielding $\theta=120^{\circ},$ or $\boxed{\textbf{C}.}$

See Also

2018 AMC 12B (ProblemsAnswer KeyResources)
Preceded by
Problem 22
Followed by
Problem 24
1 2 3 4 5 6 7 8 9 10 11 12 13 14 15 16 17 18 19 20 21 22 23 24 25
All AMC 12 Problems and Solutions

The problems on this page are copyrighted by the Mathematical Association of America's American Mathematics Competitions. AMC logo.png